Monday, March 16, 2009

blog - What I want to say

I finished a book "Leadership On the Line", which is a quite wonderful book! So, I'd like to first describe what I learned and will learn from this book on this blog. After that, a grand plan for my career, my commitment for rock-climbing, the relation between Barack Obama and me, Politics and History, future dreams, Accounting and IT, and so forth, I will mention here.

Sunday, March 15, 2009

blog - Terrible Weight

My weight is not good. I've been trying to loose weight below 59kg, but still it's over 61kg (sometimes 60kg marked). Well, that's because maybe more muscle is equipped relatively than before, but in any case this figure is unacceptable. To win the coming competition, 58kg body must be earned.

Yesterday and today, I took alcohol. That should be blamed by the god of rock-climbing. Also I ate some bunch of meat (pork) under the urgent appetite of meat. To be healthier and live long, meat must be avoided as much as possible. On non-carbohydrate days, then what I should take in?

Well, rock-climbing challenges me in terms of nutritional excellence. I must overcome any unnecessary food habits. My physical body must be like iron. Fat should be gone. And at the same time, my spirit and passion should be energetic all days.


A challenge is coming.

Tuesday, March 10, 2009

blog - Chanllenges: Investment

I lost a lot of money in the process of the financial crisis. Actually, I don't loose anything though, my once huge profit is totally gone. But know this the world. I never abandon hope.

So, at the beginning of the year, I put this area's activity onto my resolutions. I'll rebuild my weakened portfolio (my portfolio is constituted mainly by Chinese stocks) by intensively evaluating bargain priced stocks.

And I still have an ambitious target that I will make 100 million yen through investing. That might have been a likely possible one, but at this very tough time though, I still remain faithful in the value investment method and the true meaning of capitalism.


I hold faith in the basic mechanism of money.

Monday, March 09, 2009

blog - Challenges: Leadership

A business trip I had in August 2008 made me think of leadership. I met a leader who was speaking about leadership and the life of strength. I was attracted and moved by him. I realized the sense of leadership.

One who are in his thirties should focus on honing his leadership, that's my opinion. At an organization, at a some group that you belong, at a PTA (Parents T... Association), you need leadership at a time when you want change. When you want to change something to its rightful place, when you want to make something better.

Leadership is what you need to get in to yourself. What if you don't wear this ability at work, with your family or with your friends? You should. You'd better. You must. And You can attain your leadership through the activity that you are willing to learn something new, something tough. A danger makes you tough, a greater leader. This should be kept in your mind.


Leadership, I should have.

Friday, March 06, 2009

blog - Challenges: Service

Obama opened up my mind. The sense of service and sacrifice is my target for 2009 and for my life. That's required in a today's world, especially for a person who has a wish of a better world. Then, I decided at the beginning of this year that I would make a donation 10 thousand yen a month. I did...

January: 10,000 yen to WikiPedia
February: 10,000 yen to WikiPedia
March: 10,000 yen to TBD


Service has a big meaning to me. Obama proved that he engaged in public with a selfless manner. I concern about the status of our nation's politics for far too long. If you have some kind of thought that you think you should do, you do it. Following the teaching, I move forward. I will put my service into the world.


The world is.

Thursday, March 05, 2009

blog - Challenges: Change and Love!

I made a new year's resolution. That consisted of 7 parts. That is, English, Business, Climbing, Change and Love, Service, Leadership, and Investment. The most conspicuous thing is this; Change and Love...

I thought that Change is needed. Inspired by Barack Obama, the president of the United States of America, I sensed that change is also needed inside me, as well as in the world. I had undesireable things that had continued existing inside me for so long time, a portion such as fear or status quo. I must have thought my 2009 should be the year of Change and Hope. I'm going to follow the principle and his voice...

Change can bring new things to the government, the politics, and our lives. We can change our lives. We can start new things such as donating to some NPO, training our strengths, loving our friends. And Love, that can bring to us a new sense of resurgence and happiness. That surely makes us move to the next step. I'm happy to have this sense of, in the future, having a strong bond with somebody. I can see that actually.


Important thing, make you honest toward your own sense, past, present, and future.

blog - Challenges: Climbing

I'm a already good climber. I can send a 5.12a route and for bouldering, I can pass the first grade problem. Through its training, I have achieved unbelievable physical strength and mental power. My body is unmatched, and my mind is much stronger. It has been advanced according to a plan.

Now, I face a challenge. My next target is too high. That requires much more efforts and much more strengths. I need to train my body and my mind. A single arm pull up must be done easily, although I have not done yet despite three year effort. Fingers and arms I must put strength to. I will make them meet the demand of a higher grade.

Physical, mental, technique, and nutrition are keys to moving forward to the next step. I need to loose my weight below 59kg. I need to set concrete targets and achieve them for certain. Yes. I will have a competition in April, so I'm gonna win the first prize, making a revenge of the previous competition where I won the second prize.


Surely, I will devote myself to the rock-climbing area.

Wednesday, March 04, 2009

blog - Challenges: Business

I started a job-related activity (searching a new job) at the start of February. I visited several job consultant agencies, and applied for several opportunities. I failed once a position from one of the biggest software vendors in the world. The position fascinated me in terms of the product (the great CRM package) and the challenging work (Program Manager at R&D department). 

The failure made me disappointed, but soon I recovered and realized that the position requires a pure technical person. I have preference toward technologies of course, but it's not the one that meets the demand of the world class R&D position. For this several years, I've been putting a lot of efforts in other than technical area, such as business and English. So, I can accept the disappointed result as the world reflects truth.

But my journey's still going on. I'm now taking several opportunities ranging from Technical Support Engineer and Internal Project Manager to Project Manager at another largest IT vendor. Noticed by the failure for the previous opportunity, I started honing my technical skills once more. I'm reading a pro C# book, reading a network architecture, and preparing for my new project (which is that I build my own new service on the Internet).


A challenge makes my stronger and more enthusiastic.

Monday, March 02, 2009

blog - Challenges: English

As you know, English has been one of my biggest challenges in my professional life. As Japanese, I always feel learning English is quite tough for us. Lately, the irregularity of pronunciation annoys me so much! (e.g. why the intonation is different between offer and occur??)

But, the situation is changing. The hard push is making an impossible thing possible. Attaining 690 points on GMAT, I'm getting to be better off little by little, day after day. And these days, many times I am judged fluent in English, although it's far from native's.

Time and again, I struggled and depressed myself by the fact that I had not achieved my desirable target. Time and again, I felt powerless, helpless, and incompetent. But always, undaunted hope and unflinching determination have lifted me up from the darkness. Always.


Then I can conclude. I always struggle, fight, and accomplish. Something I accomplish.

Sunday, March 01, 2009

Activity, job related

I've started job related activity since the beginning of February. I failed an opportunity, which was what I thought I'd really liked to do for for the first time. But understand, I'll never ever give my dream up for a petty expedience sake.

Through the process, I came to make firm my career plan. I realized what I truly want to do. Guided by the speech spoken by the greatest president ever, I found myself in a position to pursue my desirable happiness.

I had been feeling that there would be a nicer place for me. Rather, there would be a place where I exert more leadership, more courage, more technical excellence, and more enthusiasm. Feeling those kinds of thought, I decided something. I'll change my job again.

A life itself is amazing. A challenge itself is exciting. But the most important thing is what you can make realize in your life, what you do as much as possible in your life, what you put on your life. There must be various options in your life, but the range of possibility should be focused, and you need to begin your own journey especially in your thirties.


That's what I began in my thirties.

Saturday, January 17, 2009

"The Leadership Challenge"

This is a kind of textbook. I can't see there any passion or any so-called big enlightenment, but this type of book brings to us a benefit of organizing thoughts. Here is the list of elements of how leadership works.

The Five Practices and Ten Commitments of Leadership
1. Model the Way
    - Clarify values by finding your voice and affirming shared ideals.
    - Set the example by aligning actions with shared values.
2. Inspire a Shared Vision
    - Envision the future by imagining exciting and ennobling possibilities.
    - Enlist others in a common vision by appealing to shared aspirations.
3. Challenge the Process
    - Search for opportunities by seizing the initiative and by looking outward for innovative ways to improve.
    - Experiment and take risks by constantly generating small wins and learning from experience.
4. Enable Others to Act
    - Foster collaboration by building trust and facilitating relationships.
    - Strengthen others by increasing self-determination and developing competence.
5. Encourage the Heart
    - Recognize contributions by showing appreciation for individual excellence.
    - Celebrate the values and victories by creating a spirit of community.


That's it.

Monday, December 15, 2008

essay - GMAT: Argument 38

The following appeared as part of an article in the business section of a local newspaper:

Motorcycle X has been manufactured in the United States for more than 70 years. Although one foreign company has copied the motorcycle and is selling it for less, the company has failed to attract motorcycle X customers?some say because its product lacks the exceptionally loud noise made by motorcycle X. But there must be some other explanation. After all, foreign cars tend to be quieter than similar American-made cars, but they sell at least as well. Also, television advertisements for motorcycle X highlight its durability and sleek lines, not its noisiness, and the ads typically have voice-overs or rock music rather than engine-roar on the sound track.

Discuss how well reasoned . . . etc.

Answer:
The author of the article asserts that the reason why the company selling a similar motorcycle to Motorcycle X fails to gain popularity is not because the product, which is a copy product of Motorcycle X, doesn't produce exhaust sound. Supposedly, only the characteristic that makes different from Motorcycle X is the sound. The reasons that the author introduces are, first, that the difference in noise doesn't make sense in automobile, and second, that the advertisement of Motorcycle X doesn't appeal any noisy characteristics. His argument, however, has serious flaws, and thereby it fails to convince the readers of the newspaper.

First of all, the author fails to establish connection between the reasons that he cites and the unpopularity of the product that the foreign company manufactures. That is, the symptom in cars doesn't have a direct link with the one in motorcycles, and the fact that the Motorcycle X's advertisement doesn't stress the sound nature doesn't necessarily mean that the induction that the lack of similar sound of the counter product is denied. Therefore, to strengthen the argument, the authors should have showed a direct causal relationship between his premises and the symptom.

The biggest problem from a writing structural point of view is that the article lacks a conclusion. He says that the possible cause is not a good explanation for the situation, but he doesn't show any reason that he thinks of as a major cause. He should have at least mentioned his opinion at the article.

In conclusion, his reasoning is weak because of the reasons that he cites, and above all, his reasoning lacks the conclusion, and thereby it is not considered a good argument.

essay - GMAT: Argument 37

The following appeared in an article in a college departmental newsletter:

Professor Taylor of Jones University is promoting a model of foreign language instruction in which students receive 10 weeks of intensive training, then go abroad to live with families for 10 weeks. The superiority of the model, Professor Taylor contends, is proved by the results of a study in which foreign language tests given to students at 25 other colleges show that first-year foreign language students at Jones speak more fluently after only 10 to 20 weeks in the program than do nine out of 10 foreign language majors elsewhere at the time of their graduation.

Discuss how well reasoned . . . etc.

Answer:
The article concludes that the method of foreign language instruction developed by the professor of Jones University is superior to any other programs that other universities have. The reason that the argument stands on is a study conducted at 25 universities showing that students at Jones have abilities of speaking more fluently than other students at other universities. The reasoning, however, has several serous flaws, and thereby fails to make it convincing.

First, and most important, the argument completely ignores a significant difference between a nature of the promoting program and the others, which is considered a major defect of the study. That is, the program that the article supports has a kind of special treatment which is to live abroad for 10 weeks. Supposedly, not all other programs have this kind of special investment that typically costs a large amount of money. This means that the study represents just a result of the students' proficiency, not a superiority of the programs.

Second, we need to scrutinize what proficiencies the study was intended to figure out. The article says that the students can speak more fluently, but what is the definition of speaking fluently? Does the study not focus on a grammatical aspect or not judge the preciousness? Since the article doesn't show this kind of information, we can't measure the level of the students at Jones, and judge whether the instruction method is superior or not.

Finally, we must cast a doubt on reliability of the survey. Since the article doesn't provide any specific information concerning the method of the survey or the interviewed group, we can't ensure whether the survey was properly conducted. What if students at Jones University were all from the country that the surveyed foreign language is spoken? What if students at the other universities take the foreign language courses as a second chose language class, which means that they don't fully focus on the classes? Unless the article shows reliability of the study, we are not in a position to approve that the instruction method has superiority.

In conclusion, for reasons that I mentioned above, the author of the article fails to convince the readers to regard the instruction method as superior one. To make the argument convincing, the author must at least show the detailed information of the study and then perform reasonable comparison that can make the readers find the method superior.

essay - GMAT: Argument 36

The following appeared as part of a recommendation from the financial planning office to the administration of Fern Valley University:

In the past few years, Fern Valley University has suffered from a decline in both enrollments and admissions applications. The reason can be discovered from our students, who most often cite poor teaching and inadequate library resources as their chief sources of dissatisfaction with Fern Valley. Therefore, in order to increase the number of students attending our university, and hence to regain our position as the most prestigious university in the greater Fern Valley metropolitan area, it is necessary to initiate a fund-raising campaign among the alumni that will enable us to expand the range of subjects we teach and to increase the size of our library facilities.

Discuss how well reasoned . . . etc.

Answer:
The author of the recommendation says that Fern Valley University should entice contributions from alumni and invest the money in increasing the number of subjects and the size of its library in order to tackle a recent decline of the number of admissions and enrollments. The premise that the author stands on is a voice of the students saying that teaching is poor and the library resources are inadequate. His reasoning, however, has a significant flaw.

His flaw is that his proposal won't solve issues that the university has. That is, first, whereas the students appoint poor teaching, his proposal suggests expansion of the number of subjects. Will expanding the number of subjects solve students' dissatisfaction toward poor teaching? Probably not. It's precisely because students don't mention the number of subjects as their complain, and mainly because the heart of the problem lies with other places. Improper methods of teaching, a poor system of sharing information among faculty about the status of students, or simply, unhappy and inertia teachers complaining low salaries might be the problem for instance. Therefore, it's plausible that his proposed solution won't achieve his target.

Although his second proposed solution is to increase the size of the library facilities, this will also fail to reach the heart of the problem with a high possibility. The students mention inadequate library resources, not the size of the library. They possibly concern about the balance of genre, or maybe the obsolescent books. Therefore, simply increasing the size of the library and books won't solve the problem with a high prospective.

In conclusion, his proposal is inadequate for combating problems that the university has been suffering from. To make a convincing proposal, the author, in the first place, needs to analyze the root causes again and develop solutions that directly reach the heart of the problems.

Friday, December 05, 2008

essay - GMAT: Issue 42

The best way to preserve the natural environment is to impose penalties?whether fines, imprisonment, or other punishments?on those who are most responsible for polluting or otherwise damaging it.

Discuss the extent to which you agree or disagree with the opinion expressed above. Support your point of view with reasons and/or examples from your own experience, observations, or reading.

Answer:
I don't agree with the remark saying that punishment is the best way to preserve natural environment. I, surely, think of it as a way of achieving preservation of natural environment, but it is just one way, not the best way.

Punishment is not a way to reach a heart of problems. Instead, the best way to preserve natural environment is, I believe, to educate people, or in other words, get people fully aware of the imminent troubles. Admittedly, some people restrain themselves from damaging environment if they know they are fined by doing so. What we should recognize is, however, that some people will come up with a sort of loophole to avoid the punishment. And, needless to say, awakened people don't need such punishment in order to discipline themselves. Therefore, I contend that punishment should not be focused; what should be focused is to enlighten people.

Moreover, difficult and realistic problems will appear if the strengthened punishment policy were to be introduced. How to define the scheme of punishment? How to detect the deficiencies? I know that it's relatively easy to measure the amount of noxious wastes from industrial plants, but I bet that it's quite difficult to do so for households. How we can reduce unnecessary wastes and protect our planet is up to people's awareness. Wars have almost disappeared because of people's awareness of the dangerousness and meaninglessness. Racial descrimination has almost been conquered because of people's awareness of what is right, not the penarties.

In sum, although I agree with the opinion that imposing penalties is one of approaches to preserve natural environment, my belief tells me what is more important; people's awareness toward the pressing deteriorating future that we should prevent from happening.

essay - GMAT: Issue 41

With the increasing emphasis on a global economy and international cooperation, people need to understand that their role as citizens of the world is more important than their role as citizens of a particular country.

Discuss the extent to which you agree or disagree with the opinion stated above. Support your views with reasons and/or examples from your own experience, observations, or reading.

Answer:
The remark is oversimplified. Which is important between a role of being a global citizen and a role of being a citizen in a local community varies with situations and contexts where the citizenship is focused.

It's surely true that the world is connected, globalism surges rapidly, and we face unprecedented crises that require international cooperation. The world is connected in terms of both logical distances and political bonds. The Internet and the end of the Cold War make it happen. As a result, globalism becomes the most remarkable aspect in living in the contemporary era. Besides, such as the recent financial crisis that happened from a bankruptcy of Leman Brothers in U.S., crises that we have never seen before urge us to combat them worldwide cooperatively. This movement can not stop, and thereby the importance of being a global citizen increases according to the time being.

On the other hand, the importance of being one of members of local communities and countries never diminishes. Rather, according that globalism advances, the importance grows. To take an important role in global contexts, you must know your country and represent it as a kind of an envoy. To make the world better, you should first contribute to your local community and serve it as a volunteer. And, the development of your local community and country makes it happen that the world consisting of each country enjoys the prosperity.

To sum up, the remark saying that a role of being a global citizen is more important than a role of a citizen in own countries is completely nonsense, because the argument varies with the contexts, and ultimately both of them are important.

essay - GMAT: Issue 40

It is difficult for people to achieve professional success without sacrificing important aspects of a fulfilling personal life.

Discuss the extent to which you agree or disagree with the opinion stated above. Support your views with reasons and/or examples from your own experience, observations, or reading.

Answer:
I, in general, agree with the remark that a certain amount of personal sacrifice is required to achieve success in the professional life. Surely, not-so-hard achievements can be easily attained without any sacrifices. However, for a great success or an unprecedented achievement, I have reached this conclusion by general deduction and personal observations and experiences.

My principle says that if you want something difficult to achieve, you need to sacrifice something commensurate with what you want. I lately read an article about Codolesa Rice, who is the secretary of state of the United States of America, a young black brilliantly smart woman, which says that she pitched in very hard to achieve the success studying in schools twice as much as others. Besides, we can easily observe that all people who are known for great achievements in history actually sacrificed something, and thereby they spent their lives completely different from others. Edison, an inventor of an electricity bulb, achieved his success saying 99 percent of perspiration is important.

My personal experience also supports the remark. When I achieved something such as winning a big deal in business, I always sacrificed a part of my personal life. I refrained myself from drinking alcohol or going out with my friends. I shortened my sleeping time instead to devote in study. And, I'm sure that I will in the future sacrifice something to achieve my targets. It's because I know that without the sense of sacrifice people cannot realize their dreams at all.

At the same time, we don't need to sacrifice what we can do without sacrificing. I mean, we don't necessarily need to leave our families behind in order to make success in professional lives. We don't need to give up our important activities such as exercising or volunteering at any cases. We can see all the President of the United States of America had and have their families and spent a certain amount of time with them. We can see many competent professionals have their hobbies or their volunteer work. Although a great power comes with a great responsibility, it's not necessarily the case that we always give up our important activities in order to achieve great successes.

To sum up, I generally agree with the claim that professional success requires personal sacrifice, although there are exceptions that we can enjoy both of them at the same time.

essay - GMAT: Issue 39

Courtesy is rapidly disappearing from everyday interactions, and as a result, we are all the poorer for it.

From your perspective, is this an accurate observation? Why or why not? Explain, using reasons and/or examples from your own experience, observations, or reading.

Answer:
I don't think that courtesy is disappearing and we are getting poorer for it. Courtesy is still alive and is thriving as a depth of our civilization advances, and people are getting more polite and compassionate little by little. I'll show my belief toward courtesy and mention how it has been changing through the time.

Courtesy is a kind of oil which makes our interactions better. When people put oil to a machinery device, the machine works better, for the oil makes the cogs and levers inside the machine smooth. Courtesy works as oil does. Anywhere people communicate with others, courtesy is needed to have good communication. People meet their neighbors in the morning, and they greet each other with courtesy. People have meetings with their business partners, and they negotiate and discuss with courtesy in order to build a win-win relationship which is only a way to make the deal thriving. If courtesy were not there, people hate each other and as a result, they devastate themselves as well as their opponents. And courtesy has been developed according that the time goes by. People have acquired it through the development in their civilization and humanity, because they knew that it was critically necessary to develop a good shape of the world.

At the same time, as the speaker concerns, advance of human beings brings bad aspects in terms of communication. The Internet brought us evolutionary methods to communicate, but it also brought us fewer opportunities to communicate with others in person. Industrization brought us exquisite convenience to live out, but it also brought us solidarity which is caused by the useful lives. And as a result, it may be said that people face a possibility that they undermine their ability to communicate with the others. I believe, however, that good aspects of our evolution overcome bad aspects of it. Therefore, people continuously develop their courtesy as a tool of better communication and as a result, people enjoy their development of humanity and prosperity.

To sum up, the opinion that courtesy is diminishing and people are getting poorer at courtesy is apparently exaggerated. It is just a small aspect of our whole nature. In my belief, people continue developing their ability of communication including courtesy and will enjoy better lives with courtesy.

Thursday, November 27, 2008

essay - GMAT: Issue 38

Since science and technology are becoming more and more essential to modern society, schools should devote more time to teaching science and technology and less to teaching the arts and humanities.

Discuss the extent to which you agree or disagree with the opinion stated above. Support your views with reasons and/or examples from your own experience, observations, or reading.

Answer:
I totally disagree with the opinion that teaching science and technology in schools is more important than teaching arts and humanities today. The speaker premises that contemporary societies require science and technology more and more, but arts and humanities, I believe, are also vital in today's society.

I agree with the statement that teaching science and technology is important, but it is not what can be compared with the importance of teaching arts and morals. Nowadays, it is said that the world is connected by our science in terms of the emergence of the Internet and the progress of globalism. At the same time, however, we can also say that arts and the sense of humanity can connect every place in our world with each other. In my belief, as the level of civilization advances, interests of people expand toward various areas such as arts, music, and sports. In this sense, although science and technology are necessary to teach in schools, it is not necessarily the case that teaching arts and ethics can be underestimated.

People can develop their cultural senses through being familiar with arts. Visiting art museums and enjoying sophisticated paintings, for instance, is a good way to cultivate spirit. In the sense, teaching arts and making students to equip themselves with an ability of enjoying arts is essential in today's world.

Humanity is an indispensable piece in living in today's tightly connected world. After 9.11, it can be observed that the world became a place difficult to live safely, but if I could find out something that could be clue to solving the intricate world, I would say that it is education of humanity. People have to learn the way of living with people who have different religion or belief. People have to learn the sense of compassion in order to remake the world a safe place to live. Teaching humanity is one of the most important factors to make it possible.

To sum up, I believe that teaching arts and humanities are still important, or getting more and more important, in today's world, while science and technology are also necessary. Arts and humanities help people spend better lives and connect themselves with each other.

essay - GMAT: Issue 37

Businesses and other organizations have overemphasized the importance of working as a team. Clearly, in any human group, it is the strong individual, the person with the most commitment and energy, who gets things done.

Discuss the extent to which you agree or disagree with the opinion stated above. Support your views with reasons and/or examples from your own experience, observations, or reading.

Answer:
I totally disagree with the opinion that working as a team is much inferior to the power of individuals. Working as a team is the most essential for almost all enterprises, even though the importance of individual ability remains unchanged. I'd like to describe the importance of the team below, mentioning differences in various industries and the size of corporations.

Admittedly, the importance of the role of individuals varies with the industries. That is, enterprises providing a certain professional service such as law services and medical cares might not need the strength of forming team as much as the other types of business. A super intelligent attorney can bring big profits to the company, because courts are places where a few attorneys work at any given time. Almost all enterprises, however, need a power of teamwork, because almost all work that the company needs to perform requires a certain amount of employees; in other words, an individual can affect a limited influence. How does a factory of automobiles operate? How does a product such as a candy bar appear in front of customers? Tens and hundreds, or thousands, people need to work as a team to perform a very single task in the business function. In the environment, what an individual can do is limited, and above all, emphasis on individualism should not disturb the sense of cooperation that is essential to form a great team.

A size of the corporation might also be one factor of determining the degree of influence of teams. At a small business, certainly, a competent person can make a big change on the company, because the individual can do a certain work affecting the outcome of the business, and the influence of the individual reaches out a certain degree in the company. On the other hand, at big corporations such as Fortune 500 companies, the range that an individual can affect is limited. Thus, for the employees, it becomes important to work as a team rather than tend to exert his individual influence. From my personal experience, I have worked for both sizes of corporation. At a middle-sized company, I could enjoy my personal significance, while at a big-sized company that I currently belong to, I know the limitation of my personal influence and try to maximize the power of the team that I belong to.

In conclusion, my belief shows that working as a team is much more important than exerting personal influence at any organizations. Although the degree depends on the type and size of the business to some extent, how we work effectively and efficiently as a team will make a big difference for all companies.

essay - GMAT: Issue 36

Businesses and other organizations have overemphasized the importance of working as a team. Clearly, in any human group, it is the strong individual, the person with the most commitment and energy, who gets things done.

Discuss the extent to which you agree or disagree with the opinion stated above. Support your views with reasons and/or examples from your own experience, observations, or reading.

Answer:
I totally disagree with the opinion that working as a team is much inferior to the power of individuals. Working as a team is the most essential for almost all enterprises, even though the importance of individual ability remains unchanged. I'd like to describe the importance of the team below, mentioning differences in various industries and the size of corporations.

Admittedly, the importance of the role of individuals vary with the industries. That is, enterprises providing a certain professional service such as law services and medical cares might not need the strength of forming team as much as the other types of business. A super intelligent attorney can bring big profits to the company, because courts are places where a few attorneys work at any given time. Almost all enterprises, however, need a power of teamwork, because almost all work that the company needs to perform require a certain amount of employees; in other words, an individual can affect a limited influence. How does a factory of automobiles operate? How does a product such as a candy bar appear in front of customers? Tens and hundreds, or thousands, people need to work as a team to perform a very single task in the business function. In the environment, what an individual can do is limited, and above all, emphasis on individualism should not disturb the sense of coorporation that is essential to form a great team.

A size of the corporation might also be one factor of determining the degree of influence of teams. At a small business, certainly, a competent person can make a big change on the company, because the individual can do a certain work affecting the outcome of the business, and the influence of the individual reaches out a certain degree in the company. On the other hand, at big corporations such as Fortune 500 companies, the range that an individual can affect is limitted. Thus, for the employees, it becomes important to work as a team rather than tend to exert his individual influence. From my personal experience, I have worked for both sizes of corporation. At a middle-sized company, I could enjoy my personal significance, while at a big-sized company that I currently belong to, I know the limitation of my personal influence and try to maximize the power of the team that I belong to.

In conclusion, my belief shows that working as a team is much more important than exerting personal influence at any organizations. Although the degree depends on the type and size of the business to some extent, how we work effectively and efficiently as a team will make a big difference for all companies.

essay - GMAT: Argument 35

The following appeared as part of a plan proposed by an executive of the Easy Credit Company to the president:

The Easy Credit Company would gain an advantage over competing credit card services if we were to donate a portion of the proceeds from the use of our cards to a well-known environmental organization in exchange for the use of its symbol or logo on our card. Since a recent poll shows that a large percentage of the public is concerned about environmental issues, this policy would attract new customers, increase use among existing customers, and enable us to charge interest rates that are higher than the lowest ones available.

Discuss how well reasoned . . . etc.

Answer:
The author, one of executives of the Easy Credit Company, proposes that the company donate a certain portion of its profits to a well-known environmental organization in order to boost its sales. His logic is that if the company were to put the organization's logo mark on the company's credit card, it would be able to increase the sales through attracting new customers and making current customers use their credit cards more. The assumption on which he bases his argument is that a phenomenon that people highly worry about environmental issues was observed in a survey, and thereby they would be attracted by his new marketing strategy. His reasoning, however, has a serious flaw, and consequently his proposal would not be accepted by the president.

The flaw is that he fails to reasonably connect the increase of credit card usage with people's awareness toward environment. As the poll that he cites says, people certainly become more conscious to environmental matters. More and more people use their own shopping bags instead of plastic bags that they can get at stores for free. More and more people are about choosing fuel-efficient vehicles rather than heavy and sizable SUVs that hugely consume oil and emit green gas. However, all merchandises or services can not necessarily harness this trend. Admittedly, famous organization's logo mark on a credit card might attract a few people, but it can be considered that most of consumers don't pay any attention on how their credit card company donates some portion of its profits to an environment organization. Therefore, his reasoning is too weak to make it convincing.

To strengthen the argument, the author needs to show clear evidence that supports his conclusion. It might be a customer awareness survey or a case study of the other company that adapted the similar strategy. In doing so, his argument becomes convincing, and he will be a position of persuading the president.

In conclusion, although his proposal might attract a little bunch of consumers who strongly have environmental concern, since it fails to show any apparent link between a logo mark of environmental organization on the credit card to a sales increase, his argument is weak.

essay - GMAT: Issue 35

All citizens should be required to perform a specified amount of public service. Such service would benefit not only the country as a whole but also the individual participants.

Discuss the extent to which you agree or disagree with the opinion stated above. Support your views with reasons and/or examples from your own experience, observations, or reading.

Answer:
I generally agree with the statement saying that all citizens should engage in a certain amount of public service. I also support the idea that engaging in public service benefits both the public and the individual. But, at the same time, I'd like to mention a necessary mechanizm that expedites volunteering spirits.

The first reason why I emphasize the importance of engaging in public service is that I believe that through the experience a person can acquire a broad sense of comtemporary society.

The next reason is that people benefit themselves through engaging in such activity as the statement asserts.

What I'd like to mention is that a society needs a certain mechanism of boosting the spirits of volunteering.

To sum up, I agree with the statement as a whole.

essay - GMAT: Argument 34

The following appeared in an editorial from a magazine produced by an organization dedicated to environmental protection:

In order to effectively reduce the amount of environmental damage that industrial manufacturing plants cause, those who manage the plants must be aware of the specific amount and types of damage caused by each of their various manufacturing processes. However, few corporations have enough financial incentive to monitor this information. In order to guarantee that corporations reduce the damage caused by their plants, the federal government should require every corporation to produce detailed annual reports on the environmental impact of their manufacturing process, and the government should impose stiff financial penalties for failure to produce these reports.

Discuss how well reasoned . . .etc.

Answer:
The author asserts that the federal government should require corporations to submit annual reports regarding environmental dameges that the manufacturing process in their plants could cause. Besides, if a corporation fails to do that, the government should impose financial penalty on the corportion. The assumption that he bases his argument on is that environmental damage is caused by corporate management's unawareness of their plants' manufacturing processes harming the damage. His argument, however, has serious flaws and thereby is weak.

First, the solution that he deduced from his argument is too indirect to accomplish the ultimate target, which is to preserve environment. I mean, just requiring corporations to submit annual reports on their manufacturing processes is not enough to protect environment. For instace, despite corporate management's awareness of the manufacturing process, without conclete corrective actions such as improving the plant's desposing process, submitting reports makes no sense.

Second, although his plan says that corporations need to submit the report annually, I believe that it is not enough from a time point of view. I mean, in today's fact-changing world, the timing can't catch up most of changes that occur on corporations' manufacturing processes. For instance, during the period, a lot of plants could be built and they could emitted huge hazardous byproducts.

In conclusion, he fails to make his plan convincing for reasons that I mentioned above. To strengthen the idea, he must clarify a clear link between his plan and the outcome (the outcome must be targeted on environmental preservation), and consider more appropriate timeframe that the government requires any action of corporations.

Wednesday, November 19, 2008

blog - About My Future

I performed my "final" TOEFL and GMAT. I also got the results. TOEFL is 102. GMAT 610 (AWA: 5.0).

This is bad, so I cannot go to top MBA schools with this scores. I was disappointed. But, I decided to take one more exam in December. Besides, I decided to knock on a door of MBA consultant agency. And I decided to start further applying process. I need to set off writing essays.


I need do.

Tuesday, November 04, 2008

blog - Judgement Day Come! (Obama 2008)


(Translated Speech Movie. Just One More Day.)

I know it will come. But this day makes me nervous until it decides.

One thing is clear. America has been excited about this election. Many, many people cast their ballots at this histroic election. Enthusiasms come.


Yes, we can.

essay - GMAT: Issue 34

People are likely to accept as a leader only someone who has demonstrated an ability to perform the same tasks that he or she expects others to perform.

Discuss the extent to which you agree or disagree with the opinion stated above. Support your views with reasons and/or examples from your own experience, observations, or reading.

Answer:
I, in general, agree with the statement saying that people are likely to accept as their leader a person who can perform the same job that he or she wants others to perform, although some complementally explanations are required.

My belief in leadership is aligned with the statement. Only a person who can convince others that he has an ability of performing the job well can obtain their supports. In other words, a person who doesn't have any knowledge or understanding in the specific area that he belongs to can't exert leadership, since his colleagues and subordinates don't show their respects to him. In my country, Japan, this is tremendously true. Only great artisans can have apprentices. Only business persons who have great innovative minds can be accepted as great leaders.

The statement, however, needs some corrections. First, a person who can't perform a certain job anymore can also exert leadership. No wonder that almost all coaches in professional sports such as baseball or football don't have the same ability to play the sport as the players. Yet it is possible that they can exert their leadership. Why? It's because they have deep knowledge and experience in the area and as a result they can apply them to the present situation. Moreover, look at industries where technological changes frequently happen, say, Information Technology industry. Most of senior level managers don't know any specific products such as Windows Vista, which is the latest dominant operation system working on personal computers, but they can be respected by their colleagues or subordinates because of their ability of quickly catching a concept of state-of-the-art technologies and their fundamental understandings toward basic principles.

Finally, I would like to mention serious situation that can be happened in case an inappropriate person is in a leader's position. Remember many MBA, which stands for Master of Business Administration, graduates who doesn't have specific knowledge or experience failed to operate the business that had been acquired in M&A, Merger and Acquire. Since they could not exert leadership, they failed.

To sum up, my belief toward leadership matches the statement as a whole, although some complements are required.

essay - GMAT: Argument 33

The following is part of a business plan being discussed at a board meeting of the Perks Company:

It is no longer cost-effective for the Perks Company to continue offering its employees a generous package of benefits and incentives year after year. In periods when national unemployment rates are low, Perks may need to offer such a package in order to attract and keep good employees, but since national unemployment rates are now high, Perks does not need to offer the same benefits and incentives. The money thus saved could be better used to replace the existing plant machinery with more technologically sophisticated equipment, or even to build an additional
plant.

Discuss how well reasoned . . . etc.

Answer:
The author asserts that the company, Perks Company, should stop providing its employee beneficial program on the ground that the national unemployment rate is high and thereby the company doesn't need to provide the program to attract new employees anymore. He also advocates that the company could use the money thus saved in other investments such as procuring new equipment or building new plants. The argument, however, has serious flaws, so that it fails to be convincing.

First, although the author cites data of national average unemployment rate, this survey can be considered to be a survey targeting all industries, not a specific industry. That is, the survey must have been conducted for businesses ranging from manufactures, and wholesalers, to retailers. And also it can be inferred that all industries ranging from department operators to oil industry were the targets. In evaluating his plan, this type of data can mislead the judgment. In this case, what is needed is data showing unemployment rate for the particular industry that Perks Company belongs to. Therefore, since the argument stands on the wrong assumption that the survey shows the unemployment rate that can be the basis for the judgment, it fails to be convincing.

Second, the argument is weak because of the fact that the risk has not been assessed in the argument. I mean, inferentially, the policy of stopping the beneficial program would cause a certain bad effect on the company. It might happen that the current employees would start thinking of changing the job. It might also happen that even those who are seeking a new job in the environment that unemployment rate is high would embrace unpreferable image toward the company. Therefore, the author should assess what kind of risk could be happened by implementing his proposal. After disclosing the risk and taking the mitigation plan in place, the proposal can obtain persuasive power.

In conclusion, the argument is weak for the reasons that I mentioned above. To strengthen the argument, the author needs to provide industry-specific statistical data and include risk mitigation plan to tackle by-products generated by implementing his plan.

essay - GMAT: Issue 33

You can tell the ideas of a nation by its advertisements.

Explain what you think this quotation means and discuss the extent to which you agree or disagree with it. Develop your position with reasons and/or specific examples drawn from history, current events, or your own experience, observations, or reading.

Answer:
I think that the quotation says that advertisements can represent the ideas of a nation. People see advertisement, TV commercials, radio ones, bulletin boards, whatever, at their homes, at offices, or on trains, and only advertisements that can attract people's attention can survive. In this sense, advertisements can show what the people prefer, what the people are interested in. I agree with this notion.

But on the other hand, no doubt that advertisements have a limitation in showing the ideas of a nation or people. For instance, in a country, there are a lot of well-manufactured TV ads promoting pregnant testers. A person cannot judge whether the people living in the country support abortion or oppose it, while he can conceive that pregnancy is a high-focused topic in the country.

To sum up, I agree partly with the quotation, but it is apparently oversimplified.

essay - GMAT: Argument 32

The following appeared as part of the business plan of an investment and financial consulting firm:

Studies suggest that an average coffee drinker fs consumption of coffee increases with age, from age 10 through age 60. Even after age 60, coffee consumption remains high. The average cola drinker fs consumption of cola, however, declines with increasing age. Both of these trends have remained stable for the past 40 years. Given that the number of older adults will significantly increase as the population ages over the next 20 years, it follows that the demand for coffee will increase and the demand for cola will decrease during this period. We should, therefore, consider transferring our investments from Cola Loca to Early Bird Coffee.

Discuss how well reasoned . . . etc.

Answer:
The author says that the company should discuss a strategy of shifting the investment from the coffee brand to the cola brand. The based argument is twofold: first, recent trends for both cola and coffee toward population ages supports the conclusion, and then a prediction that older adults will increase for the next 20 years also supports his suggestion. The argument, however, is unconvincing because of two unwarranted assumption that the argument is relying on.

The first unwarranted assumption is that the trend of consumer's preference will continue for the next 20 years. Admittedly, a trend that has lasted for the past 40 years can be considered to last continuously, but it cannot also be denied that the trend may change in the future. Therefore, the author needs to show clear information that the argument can stand on.

The second unwarranted assumption is that the author assumes that young population will decrease for the next 20 years whereas old one will increase. But the article doesn't show the information (it says old population will increase, but dome fs say about young one). It can be possible that whereas the number of older adults will increase, young people will also increase. Since the argument is based on this unwarranted assumption, the argument is weak.

In conclusion, although the reasoning seems rational and logical, in terms of basing on two unwarranted assumption, the argument is unconvincing. To strengthen the argument, and to make his business associates to consider his proposal, the author needs to provide evidence that supports the two assumptions, such as an analyst report that predicts that the current trend will last for next more than two decades and the exact demographic statistical data that shows that young population will declaim.

essay - GMAT: Issue 32

Financial gain should be the most important factor in choosing a career.

Discuss the extent to which you agree or disagree with the opinion stated above. Support your views with reasons and/or examples from your own experience, observations, or reading.

Answer:
The speaker says that financial gain should be the most important factor in choosing a career. I don't agree with him at all. I'd like to state my opinion below.

First, I believe that the financial incentive should not be the first place for those who are seeking jobs. That should be personal preference, or the sense of contribution. I mean, the sense of that I like this job is indispensable in choosing the career. If a man likes a certain job, I bet that he can continue the job and will be able to gain higher reward and higher salary in his career. On the other hand, if he doesn ft like the job although the job brings him unbelievably high payment, I bet that the situation cannot last long and eventually he will face difficulty that possibly forces him to make a serious decision, leave the job. Therefore, from this perspective, I disagree with the speaker.

Second, I believe that people should not choose the career based on the salary, because how he can gain financial reward from the job is uncertain at a time when he chooses the career. I mean, people have a limitation in forecasting the future, and the fact that he can gain high financial gain today doesn't necessarily mean the fact that he can continue gaining it for a long time. Therefore, as I mention earlier, people should act based on their inner voices: what do you like, and what do you do for the world.

Finally, although financial gain should not come first, I believe that the financial factor is very important. That can be an evaluator or a gauge of whether the job is valuable or not. I mean that, remember the law of supply and demand, if there are few people who can engage in a certain job, the payment of the job will rise up in proportion to the population. In other words, if the job that he is choosing brings him higher salary, the job can be considered a precious one and the man who can do the job can be considered a person who can contribute to the outer world. In the sense, I admit that financial gain can be a very important factor for people in choosing the work.

In conclusion, although I admit that financial gain is an important aspect for people, I believe that financial gain should not be on the first place and that people should think what they want to do first.

essay - GMAT: Argument 31

The following appeared in a newspaper story giving advice about investments:

As overall life expectancy continues to rise, the population of our country is growing increasingly older. For example, more than 20 percent of the residents of one of our more populated regions are now at least 65 years old, and occupancy rates at resort hotels in that region declined significantly during the past six months. Because of these two related trends, a prudent investor would be well advised to sell interest in hotels and invest in hospitals and nursing homes instead.

Discuss how well reasoned . . . etc.

Answer:
The author concludes that investing in hospitals and nursing homes is recommendable and those who have invested in hotels should retreat their investment as soon as possible. The reasons that he is counting on are twofold. First, the elderly people are increasing because of prolonged life expectancy. Second, regional resort hotels are devastating its sales figure in this six months drastically. His argument, however, is weak for the following reasons.

First, there's no direct causal relationship shown between the increase of elderly people and the decrease of the sales of the hotel industry. It's unreasonable to connect them, because increased elderly people doesn't necessarily mean the drop of the sales. Rather, it might happen that the increase of elderly people will contribute to the hotel industry, because, say, they have a lot of money and time. Furthermore, whether the whole population is declining or not is not mentioned. This can also be the drawback.

Second, although the author cites the data saying the sales of local resort hotels for this six months significantly plummeted, the argument is not well established. First, six month decline doesn't necessarily mean the whole year sales figure. Generally speaking, the hotel industry has seasonality. Seasonality means, the revenue widely varies with the season. Summer resorts can attract tourists in summer, but in winter the hotels would have difficulty to do so. Besides, the article doesn't mention the source of the comparison, and thereby the argument is not reasonable.

Finally, the author recommends investing in hospitals and nursing homes instead, but there's no reason to support it shown. Therefore, the readers have no way to judge whether the investment is better than the one of hotels or not. This imperfect statement seriously damages the justification of the reasoning.

In conclusion, the argument is weak for the reasons I mentioned above. To strengthen the argument, the author must do the following things at least: showing a clear link between the trend of the population and the revenue of the hotel industry, citing a survey result for the whole year not a half, and stating the premise that his recommendation stands on.

essay - GMAT: Issue 31

Companies benefit when they discourage employees from working extra hours or taking work home. When employees spend their leisure time without eproducing f something for the job, they will be more focused and effective when they return to work.

Discuss the extent to which you agree or disagree with the opinion expressed above. Support your point of view with reasons and/or examples from your own experience, observations, or reading.

Answer:
I agree with the opinion mentioned above as a whole. Nowadays, more and more companies try to reduce employees' overtime work and enhance time for employees to spend meaningful personal lives. Yet I believe that work and life have a strong bond with each other. And also, for one who has a great ambition, thinking private life as a completely separated thing from work undermines his ambition.

"Work life balance" is a hot topic for contemporary enterprises. "Work life balance" is a concept that employees should keep a good balance between work and private life. This is the very concept that the opinion says. Spending prosperous private life contributes to the effectiveness and efficiency at the office. This concept also reduces payroll costs of overtime work, while taking benefits from more productive employees. Furthermore, this concept is necessary to attract resources outside such as new graduates or job seekers. In other words, ignoring "work life balance" makes the company to be in a serious situation in terms of its image. Therefore, in the sense, companies should try to reduce overtime work and enrich employees' private lives.

On the other hand, I believe a tight bond between work and life. I mean, thinking work and private life as the completely different things is nonsense because people think about work such as a trouble that was experienced previous day or a plan to do next day when spending their time at home to some extent, and they think about private lives such as a dream that he has or family that he needs to feed when working at the office to some extent. It can be happened that a person comes up with a great idea that he can use at his laboratory while doing shopping with his wife. My standpoint is that work and life exist mutually affecting, and taking a proper balance is the essential to have successful life.

In addition, I believe that sometimes people need to take work home or work on whole weekends at the office to achieve a higher goal. Especially, young people should make themselves absorb themselves in work completely for a certain period. The experience really makes them grow in their professional lives. In my case, I believe that since I worked taking a lot of hours when young, I now have enough competence to do work in a good manner with a short timeframe.

To sum up, although I generally agree with the opinion, I'd like to stress the importance of the synergy effect between work and life and the experience of working thoroughly.

essay - GMAT: Argument 30

The following was excerpted from the speech of a spokesperson for Synthetic Farm Products, Inc.:

Many farmers who invested in the equipment needed to make the switch from synthetic to organic fertilizers and pesticides feel that it would be too expensive to resume synthetic farming at this point. But studies of farmers who switched to organic farming last year indicate that their current crop yields are lower. Hence their purchase of organic farming equipment, a relatively minor investment compared to the losses that would result from continued lower crop yields, cannot justify persisting on an unwise course. And the choice to farm organically is financially unwise, given that it was motivated by environmental rather than economic concerns.

Discuss how well reasoned . . . etc

Answer:
The spokesperson's assertive is twofold. First, he concludes that farmers who have switched from synthetic to organic fertilizers should switch back to synthetic fertilizes, because studies conducted last year show that organic farming is less productive than synthetic farming, and because the loss exceeds the investment that the farmers have done. Second, the spokesperson also concludes that synthetic farming should be chosen if an economical factor is prioritized. His argument has two serious fallacies.

The first fallacy is that although he mentions that current crop yields of farmers performing organic farming are lower than ones of farmers performing synthetic farming, this citation contains a serious error. That is, he says "yields", but the important thing for the argument is how much they earned, say, "money". For instance, if the unit price of the products from organic farming is higher than one from synthetic farming, say, because of consumer's preference, the farmers adapting organic farming may gain much more money than synthetic farmers. Thus, the spokesperson should have mentioned the revenue instead of the amount, if he can use the figure to support his conclusion.

The second fallacy is that he ignores a trend that will occur in the future. I mean, even if organic farming is currently behind synthetic farming in terms of the economical aspect, in the near future might it be the opposite. Today, many consumers become more aware of the environmental aspect, and thereby in the near future synthetic farming would be absolutely unacceptable in terms of both the environmental aspect and the economical aspect. It's because consumers may willingly pay much higher price to organic products. In the case, switching back to synthetic farming would be not only unprofitable but also unreasonable in terms of the probable need to again switch back to organic farming.

In conclusion, although the spokesperson tries to convince the audience to switch back to synthetic farming, seriously wrong argument makes it impossible.

essay - GMAT: Issue 30

Too many people think only about getting results. The key to success, however, is to focus on the specific task at hand and not to worry about results.

What do you think this piece of advice means, and do you think that it is, on the whole, worth following? Support your views with reasons and/or examples drawn from your own experience, observations, or reading.

Answer:
Althought it contains a certain level of truth, I disagree with the opinion that the key to success is to focus on the specific task on the spot and not to care about the result. The statement is oversimplified and ignores the importance of planning. I'll first show my opinion about the result, and then refute the opinion in serveral points.

The result should not be overstated, but at the same time, it should not be understated.

essay - GMAT: Argument 29

The following appeared in the editorial section of a local newspaper:

Commuter use of the new subway train is exceeding the transit company fs projections. However, commuter use of the shuttle buses that transport people to the subway stations is below the projected volume. If the transit company expects commuters to ride the shuttle buses to the subway rather than drive there, it must either reduce the shuttle bus fares or increase the price of parking at the subway stations.

Discuss how well reasoned . . . etc.

Answer:
Although commuter use of the new subway service exceeds the transit company's expectation, the one of the shuttle bus service underperforms the expectation. In order to respond to this problem, the author of this editorial article concludes that the transit company has to reduce the fare of the shuttle bus service or increase the price of parking lots near subway stations. The argument, however, has an unwarranted assumption and fallacious reasoning.

Presumably, the author presupposes that the current situation is caused by cost-related factors. That is to say, due to the high fare of shuttle bus, commuters choose instead to use their cars to reach the subway station. And, due to the low price of the parking lots near the subway station, commuters choose to use their cars. This assumption is not only unwarranted but also dangerous. First, any evidence is not shown to assume that cost-related factors cause the current situation. That is, an insufficient number of shuttle buses, for instance, may cause this situation. Or, the area coverage also may be a possible negative factor causing the number of shuttle bus users down. Also, this type of wrong assumption would cause catastrophic situation.

Assuming that the assumption, which is that the downing power is costs, is right, yet still the author misses several important factors. First, he ignores that side effect may be caused. Remember, the problem is only for commuters. If the price of the parking lots @increase, those who are not commuters would get trouble. Therefore, at least, the author needs to mention this negative side effect.

In conclusion, since the argument has an unwarranted assumption and fallacious reasoning, he fails to convince the readers.

essay - GMAT: Issue 29

Because businesses use high-quality advertising to sell low-quality products, schools should give students extensive training in how to make informed decisions before making purchases.

Discuss the extent to which you agree or disagree with the opinion expressed above. Support your point of view with reasons and/or examples from your own experience, observations, or reading.

Answer:
I generally agree with the opinion that schools should give students extensive training in how to make informed decisions before making purchases to prevent them from being cheated by high-quality advertisement selling low-quality products. Advertisement, or marketing, has a signifcantly important influence on consumers buying various products. Therefore, things as the speaker mentions can happen in today's highly media intensive world, and young people need to be trained in behaving well.

Friday, October 31, 2008

blog - BarackObamaJapan Project

The Day is coming to us.

I started engaging in BarackObamaJapan project, translating his great speeches and uploading them onto YouTube. (Please search with 'BarackObamaJapan' or visit http://jp.youtube.com/user/BarackObamaJapan)

I've been watching him have great speeches, show deep knowledge and high-spirited principles, treat his opponents with respect, and creatively lead his campaign toward the historical day that the world is yearning for.

I bet, I'm convinced, I assure, I totally believe that Barack Obama is the one who will bring us change that we have never seen before. I thank God in spending the same time with him, sharing a common belief with him, and seeing him as a model leader.

I actually knew that he would win this election when I saw him make a victory speech in Iowa on January 3rd, 2008. I knew that he would bring us significant change when I read his brilliant two books concerning his faith, his discipline, and his dream. I knew that when he had shown his unchangeable honesty, decency, and hope.

Hope! What we are looking at is hope! The miracle that we can have a great leader who exert the influence all across the world. The world has been waiting for this moment, this miracle, this great president.

Japan will also change. All new-fledged politicians will see him realize ideal politics. All people will know him bringing us a new type of politics, bipartisanship, and a new era of democracy.

The time has come! Let's enjoy it. Let's talk about it. Let's be excited by it. On November 4th, everything will change. Not only 8 year wrong politics will end, but also the dawn of new idealism will come.


The time has come!

Tuesday, October 07, 2008

essay - GMAT: Argument 28

The following appeared in a newspaper editorial:

As violence in movies increases, so do crime rates in our cities. To combat this problem we must establish a board to censor certain movies, or we must limit admission to persons over 21 years of age. Apparently our legislators are not concerned about this issue since a bill calling for such actions recently failed to receive a majority vote.

Discuss how well reasoned . . . etc.

Answer:
The author asserts that the recent increase of crimes attributes to the resent increase of violent scenes in movies, and then some corrective actions are required. He proposes two concrete actions: to establish a board censoring the violent scenes, and to limit admission to persons over 21 years of age. Besides, he sees that the legislators are not concerned about this issue on the ground that similar legislations were lately rejected. Although his reasoning is not completely imperfect, it contains several fallacies, thereby failing to convince the readers.

First of all, although he attributes the cause of the recent increase of crimes in the cities to the increase of violent scenes in movies, this argument is apparently incorrect. This is a typical "because this happens after that, that is the reason of this" logic. I mean, the true cause of the increased crime rate may be for another reason such as an influx of immigrants or surging unemployment rate. To make the argument convincing, the author should show the direct link between the increased crimes and the increased violent scenes.

Then, assuming that the violence scene is a direct cause of recent increase crimes, we can find an error in the policy of prohibiting young audiences from violent movies. If the author wants to limit admission to young people, he needs to show clear evidence that young people are source of recent crimes. Without doing that, it's unreasonable and unfair to prohibit only young people from seeing movies. This is a kind of discrimination.

Finally, the author contends that the legislators are not concerned about the issue, but this claim is unwarranted. I mean, the fact that certain legislation couldn't attain majority doesn't necessarily mean that the legislators are not concerned about it. The legislation may have had a serious flaw such as violation of constitutional right or discriminational aspect. Then, blaming the legislators for the reason is unreasonable.

In conclusion, the author fails to make its argument convincing for some reasons that I mentioned above. To strengthen the argument, the author should at least show a direct link between the recent increase of crimes and the recent increase of violent scenes in movies, for instance, by conducting scientific human behavioral research to identify the reason of increased crimes.

essay - GMAT: Issue 28

A company fs long-term success is primarily dependent on the job satisfaction and the job security felt by the company fs employees.

Discuss the extent to which you agree or disagree with the opinion stated above. Support your views with reasons and/or examples from your own experience, observations, or reading.

Answer:
In terms of my belief that success for corporations depends on people working at the company, I agree with the opinion that a company's long-term success is primarily dependent on the job satisfaction and the job security felt by the company's employees. Yet, at the same time, we should keep in mind that the degree varies with the industry to which the company belongs and the maturity level that the company's employees present. I start with my general belief connecting to this subject, and then I mention the dependency that this subject has.

First and foremost, my strong belief about company's success is that people are fundamental success factors for any organization. People can creatively produce the company's middle-term or long-term business plan. People can invent revolutionary products and implement them in the production lines. Therefore, companies should first invest in people, not in factories or a headquarters building. Then, taking a look at the opinion articulated above, I can find a stringent bond between people themselves and their job satisfaction. How does a company keep its smart, competent employees for a long time? How does a company empower or motivate its employees? The answer is simple: give them all necessary stuff, and get them to have satisfaction that makes them think to continue working for the company. In this sense, I strongly support the idea that attaining employees' satisfaction should be the first priority for companies.

At the same time, however, we need to take into account the dependency that the policy has. That is to say, the degree that an enterprise weighs the policy depends on the industry and the maturity level. First, the effectiveness that a company earns through the people is fully influenced by the industry that the company stands on. For instance, namely heavy industrial sectors may be more influenced by its equipment or patent it has. On the other hand, most of service industry companies have a heavy weight on the people. Then, regarding a maturity level, a company should be flexible about treating its people by considering whether the people can be proactive or not. I mean, for instance, if the employees are immature, or in other words, they can not perform their jobs proactively or creatively, the company may need to take a strong control on its people by, for instance, defining its business processes forcibly or limiting employees' @autorities. What I'd like to say is that a company needs to place its position for the policy on people in a flexible manner.

To sum up, I generally agree with the idea that job satisfaction is a critical factor for any organization. Yet, at the same time, I'd like to call managers f attention to make sure that the effectiveness varies with various factors as I mentioned above.

essay - GMAT: Argument 27

The following appeared in a memorandum from a member of a financial management and consulting firm:

We have learned from an employee of Windfall, Ltd., that its accounting department, by checking about 10 percent of the last month fs purchasing invoices for errors and inconsistencies, saved the company some $10,000 in overpayments. In order to help our clients increase their net gains, we should advise each of them to institute a policy of checking all purchasing invoices for errors. Such a recommendation could also help us get the Windfall account by demonstrating to Windfall the rigorousness of our methods.

Discuss how well reasoned . . . etc.

Answer:
The author, a member of a financial management and consulting firm, asserts that the firm should take a new strategy that the consultants of the firm advise the clients to check with the purchasing invoices to detect overpayment. As the reason, he cites that an employee of WIndfall, Ltd. states that the company saved approximately 10,000 dollars by checking just about 10 percent of the last month's purchasing invoices. The author also mentions that this strategy makes it possible that the firm acquires the account of Windfall as a result of showing the rigorous methods to Windfall. His reasoning is not quite wrong, but several considerations should be taken.

First, although he doesn't show the cost of detection performed by Windfall, the cost may be higher than expected. That is, say, if the detection had cost the company more than 10,000 dollars (this possibility is quite low though), the saving would have brought no value to the company. Therefore, he should have shown this kind of information necessary to judge whether the policy is reasonable or not.

Second, it is ignored that the effectiveness of this policy depends on the company applying it. I mean that the fact that Windfall could save 10,000 dollars doesn't necessarily mean that the other companies can save the same amount of money. For instance, Windfall has a serious ill-managed ordering process; the employees give the suppliers the vague information, thereby making the supplies provide wrong invoices. In this case, if a company may take in this policy, it cannot save the same amount of money, since the company already has a process to prevent overpayments.

In conclusion, although his idea seems reasonable, since the argument has uncertainty in terms of the reasons that I mentioned above, he fails to convince the readers to agree with his idea. To strengthen the argument, at the very least, he should mention the cause of the overpayment happened in Windfall.

essay - GMAT: Issue 27

Location has traditionally been one of the most important determinants of a business fs success. The importance of location is not likely to change, no matter how advanced the development of computer communications and others kinds of technology becomes.

Discuss the extent to which you agree or disagree with the opinion stated above. Support your views with reasons and/or examples from your own experience, observations, or reading.

Answer:
The speaker says that location is one of the most important determinants for business organizations and then this will continue even under the circumstance of fast-growing development of technologies including computer communications. I agree with his opinion to some extent, but I generally believe that the importance is going to diminish little by little. Also, I point out that the importance varies with the industry that the business organization belongs.

As everybody is aware, information technology is rapidly developing. Nowadays, any smooth business activity can not be performed without email, Internet sites, laptops, and so forth. Electrical commerce (E-commerce) especially affects the importance of location. E-commerce is a type of business; commercial activity through the Internet. If a company has a commerce site on the Internet, the company can sell or buy any products on the Internet. (Of course, the product is conveyed to the customer by shipping.) In this case, location is not a crucially important factor for the company. Even a company which has only the virtual shop exists. (A typical example is Amazon.com. It sells books and other gadgets on its sites and doesn't any physical store.) Therefore, under this circumstance, the importance is relatively going down for enterprises.

Also, I'd like to spell out the relationship between the importance of location and the type of business. For retail industries, such as the food service industry or a shopping center business, location is critically important for the success. If a restaurant owner starts its business at the site where nobody comes, the failure is inevitably showing up. On the other hand, for B2B companies (B2B stands for business to business), location is not necessarily important, because the frequency of transaction is less and it is general that sales person visits the customer to make a deal. Therefore, although the speaker doesn't mention, this aspect should be taken into consideration when talking about the importance of location.

To sum up, while agreeing with the importance of location to some extent, I don't agree with the claim that the importance won't change. In addition, the importance depends on the type of business.

essay - GMAT: Argument 26

The following appeared as part of an article in a magazine on lifestyles:

Two years ago, City L was listed 14th in an annual survey that ranks cities according to the quality of life that can be enjoyed by those living in them. This information will enable people who are moving to the state in which City L is located to confidently identify one place, at least, where schools are good, housing is affordable, people are friendly, the environment is safe, and the arts flourish.

Discuss how well reasoned . . . etc.

Answer:
The author of this article indicates that City L is the best choice for people considering coming to the state where the city is located because of good schools, reasonable housing costs, friendly friends, the safe environment, and the flourished arts. This reasoning, however, is very weak since necessary information is not shown.

First, the reader can not judge the city is truly good. That is, the author shows the result of the survey conducted two years ago, but necessary information such as how many cities were surveyed or what kind of people answered the survey. Therefore, the reader has a doubt on the claim.

Second, the second sentence of the article is completely ambiguous. What does the author want to say? Since the sentence doesn't contain clear conclusion such as City L has the benefits or something, the reader gets confused in how to judge its contention.

In conclusion, this argument is really poor in terms of the lack of necessary information and clear conclusion.

Sunday, October 05, 2008

blog - Vision For Week

I'm having lunch with a person who works for the other Indian company on Wednesday. I'm going to Mitake, a mountain area west of Tokyo, with my two young brothers on Saturday. Other than that, I'm fully devoting myself to rigorous routine work mainly focused on Toefl and Gmat.

Toefl, an examination gauging English proficiency, consists of four parts: reading, listening, speaking, and writing section. My previous focal point was only on the speaking section, because my score on that was quite low. But lately revealed that my listening should be improved. In order to get nearly perfect score.

Gmat, an examination gausing verbal ability in English and quantative skills, is an exam that even US people take. This means that required English ability is much higher than exams like Toeic and Toefl, which are specifically for non-native speakers. My challenge is here. To get a high score enough to get top MBA schools, I need totally to immerse myself into Gmat world.

On the other hand, I'd also like to focus on English itself. I'd like to learn "small talk" to make conversation more smooth. I'd like to learn more specialized words especially in environment, business, politics, and IT world.


I'm ready to go.

blog - My First Surfing



My first surfing was performed well! I went to Hirai and Kashima in Ibaragi Prefacture last Saturday, and I surfed for the first time in my life. That was amazing, splandish, and my back hurt.

We had also a campfire at the previous night. That was also amazing. Shining stars, shooting stars, baked mashmaros, nice guys. Everything was perfect!

I got back home at the evening on Saturday, and I saw the Vice Presidential debate held in US between Joe Biden and Sarah Palin. That was interesting, though I got asleep at the middle of the show.

I was seeking my new possibilities last week. I got to read new books, have a lunch with new colleagues, and even go surfing for the first. That was pretty good. But at the same time, I'm feeling I get to go back to my routine tasks. Toefl, gmat, and so forth.


Wonderful the world is.

Monday, September 29, 2008

essay - GMAT: Issue 26

The best strategy for managing a business, or any enterprise, is to find the most capable people and give them as much authority as possible.

Discuss the extent to which you agree or disagree with the opinion stated above. Support your views with reasons and/or examples from your own experience, observations, or reading.

Answer:
I agree that finding the best people and giving them the environment where they freely work is the best practice for any type of organization. However, a few considerations should be taken place. That is, the effectiveness varies with the industry, and the extend to which authority should be given depends on the maturity level of the organization. I'll first show my thought about the effectiveness of the policy and then reveal two aspects that should be taken into consideration.

First, I believe that finding the best people and giving them an appropriate environment is the best strategy that any corporation could take. All activities that corporations perform come from the people. All success factors depend on the people. And all machinery is invented by the people. Therefore, hiring the best people is the first strategy that any corporation should take. Besides, empowerment is a key practice in the case, especially for the best people. Various studies show that the best people need to be empowered (i.e. delegated) in order to exert their performance fully. Therefore, I totally agree with the statement that these practices are essential for any organization

What I'd like to insist is, however, that the effect is strengthened or limited by the circumstance where the organization stands. I mean, the effect is in proportion to the room that the organization has for human resources. For instance, in case the organization is a manufacturer of iron sheets, the performance of the company most depends on the physical assets such as the factory and the machinery, not on human resources. In this case, the effect is limited to the extent that the people can exert their influence. On the other hand, in case the organization is in the service industry like the hotel or the food service industry, the degree is heightened. Therefore, at the time thinking of the effectiveness of the strategy, this point should be highlighted.

In addition, the degree of given authority should be taken into consideration when thinking of this strategy. That is, giving people authority sometimes causes trouble in case the people are not ready to be given or in case the environment of the company is not appropriate for it. For instance, if the employee's maturity level is low (maturity level means the constituent's skills or proactivity, say, the professionalism), delegating power to the employees will cause unexpected catastrophe such as sabotage. Or, if the company faces serious or extremely fast-changing situation like a crisis of bankruptcy, giving the employees authority is not a good option (since the clear chain of command and the fast decision making is necessary in this case). Therefore, this aspect should be considered, when empowerment takes place.

To sum up, as a whole I support the idea that getting the best people in place and giving them authority is the best strategy for any organization, although some considerations should be taken as I mentioned above.

essay - GMAT: Argument 25

The following appeared in a memo from the customer service division to the manager of Mammon Savings and Loan:

We believe that improved customer service is the best way for us to differentiate ourselves from competitors and attract new customers. We can offer our customers better service by reducing waiting time in teller lines from an average of six minutes to an average of three. By opening for business at 8:30 instead of 9:00, and by remaining open for an additional hour beyond our current closing time, we will be better able to accommodate the busy schedules of our customers. These changes will enhance our bank fs image as the most customer-friendly bank in town and give us the edge over our competition.

Discuss how well reasoned . . . etc.

Answer:
In service industries, the quality of customer service has a critical role in enhancing the competitive advantage. The author of the memo tries to convince the managers that Mammon Savings and Loan can have a competitively strong position over the competitors through improving its customer services. His plan is to shorten customer's waiting time in the lines of teller and broaden its service window one and a half hour. His argument is generally reasonable, but for several reasons it fails to convince the readers to go about the plan.

First, and most important, the argument ignores the side effect of its improvements. That is, the service improvement plan comes with huge costs, and the author by no means mentions the aspect. Apparently, if the argument is not equipped with necessary information, the argument fails to function. In terms of this, the argument is so weak that the manager of customer service department fails to obtain agreement from his colleagues. Then, what types of cost can be considered? Above all, payroll costs would rise up much higher than expected. One of his plans is to increase open hours for customer's convenience one and a half hour, but the increase of the payroll costs is not the proportion of the increased time. In order to enhance the time window, generally, more resource than increased timeframe is required. Also, regarding one of his proposals, improving the customer's waiting time, the company needs to put much effort to cut it in half. The company, for instance, needs to set up new service desks and hire new service personnel, as well as restructure its business process maybe by using a consultancy agent. In sum, if the company implement the improvement plans, then its customer satisfaction might show the improvement. But don't forget that it would come with huge costs.

Second, he fails to show a specific approach to improve the customer's waiting time. Although he easily says that the time can be half of current one, it's easier said than done. As I mentioned earlier, it needs much effort to realize. Therefore, the manager should have mentioned his approach for the improvement.

In conclusion, it's right that improvement of customer satisfaction in service sectors considerably contributes to the increased brand image and differentiation from the competitors, but as I mentioned above, due to the lack of necessary information, the argument fails to be convincing.

Thursday, September 25, 2008

essay - GMAT: Issue 25

A powerful business leader has far more opportunity to influence the course of a community or a nation than does any government official.

Discuss the extent to which you agree or disagree with the opinion stated above. Support your views with reasons and/or examples from your own experience, observations, or reading.

Answer:
I don't agree with the opinion that business leaders can exert more influence on the course of a community or a nation than government officials. I think that government officials have advantages in terms of its power, its way, and its intention.

First, I believe that government officials have more power over the course of the community or the nation than do business leaders. The typical example is the president of the United States of America. Who can have more influence than President Bush does? He decided to start the Iraqi Invasion, not to sign Kyoto protocol, and to try to introduce 700 billion dollars bail out for Wall Street meltdown. On the other hand, Bill Gates, a founder of Microsoft, admittedly influenced the way of the world by producing a revolutionary product, Windows, but, in terms of the course of the nation, he did a little, I bet.

Second, it's clear that government officials have much more ways to exert influence on the community or the nation than business leaders do. That is, government officials have a lot of direct ways to affect the community or the nation such as constituting a new law, attending a congress, or making a speech for public. On the other hand, business leaders do not have. They can just exert indirect influence such as producing better products or advancing an opinion to government. Or, they are just privately able to attend various events such as casting a ballot or engaging in volunteer work. Therefore, as for the ways, business leaders have much less options than government officials do.

Third, I can say that if somebody wants to make the world better or change the course of the government, he would definitely choose to be a politician rather than a CEO of a corporation. I can take an example of Barack Obama, a candidate for the next president of the United States of America, my favorite politician at present. He had a choice when I graduated from university. The choice was that he would be a business person in Manhattan or would be a community organizer in Chicago. If he had chosen the way of being in the business world, he would have made big money, since he was pretty competent even at doing business. He, however, chose the way of a local community officer, although the payment was much less than one of the other job. Why did he do such a folly? The answer is simple. He just wanted to influence on the community, just wanted to improve people's quality of live, and just had an intention of changing the world. He did and will do in the United States of America.

To sum up, I disagree with the statement saying that business leaders have more influence on the community or the government than government officials do. It's because government officials have more power, because they have more ways, and because they have clearer intentions to change the course of the community or the government than business leaders do.

essay - GMAT: Argument 24

The following appeared in a speech delivered by a member of the city council:

Twenty years ago, only half of the students who graduated from Einstein High School went on to attend a college or university. Today, two?thirds of the students who graduate from Einstein do so. Clearly, Einstein has improved its educational effectiveness over the past two decades. This improvement has occurred despite the fact that the school fs funding, when adjusted for inflation, is about the same as it was 20 years ago. Therefore, we do not need to make any substantial increase in the school fs funding at this time.

Discuss how well reasoned . . . etc.

Answer:
The speaker, a member of the city council, asserts that no additional funding toward Einstein High School is required because the school has not needed an increase of the funding for 20 years to date and still has been improving its service. And he cites statistical data showing what percentage of the students advance to a college or university in order to support his conclusion. His conclusion, however, completely unreasonable because his reasoning contains several serious fallacies.

First, he assumes that the increase of proportion of students going on to attend a college or university is owing to the efforts of the school in improving its educational effectiveness. This assumption, however, is unwarranted because there is no clear evidence showing a direct link between the effort and the increase. That is, the increase might have been just due to a nationwide trend of increased sense toward higher education. To make it improve, the speaker should show the nationwide average of the rate at least, and should show a clear causal relationship between the effort and the increase.

Then, although the speaker reasons that there has been no increase of funding for 20 years at the school, this reasoning has big room to be criticized. That is, no increase of funding to the school doesn't necessarily mean no increase of investment in improving the service. For instance, maybe the number of the students had decreased, say, two-thirds for two decades. In the case, the school had been able to utilize its remained funding to improve its service. Therefore, his assumption that there was no fund increased is unwarranted. And this weakens the conclusion seriously.

Finally, he ignores the fact that the past trend does not necessarily ascertain the future trend. That is, his reasoning stands on the logic that something wasn't needed in the past so it must be the same in the future. This reasoning, however, may result in a serious situation. For instance, nowadays the development of information technology hikes up all around the world, so more schools have started investing in such kind of equipment such as computers or broad band Internet connection. In such case, unless the school invests a certain amount of money in the technology, the school cannot follow the trend, thereby making the students less competitive. Therefore, the speaker should have considered other factors that need to be considered when talking of the future.

In conclusion, since the speaker's reasoning has serious flaws, his conclusion that there fs no need to increase the amount of funding to Einstein High School is completely problematic.